LSAT and Law School Admissions Forum

Get expert LSAT preparation and law school admissions advice from PowerScore Test Preparation.

 Administrator
PowerScore Staff
  • PowerScore Staff
  • Posts: 8916
  • Joined: Feb 02, 2011
|
#41256
Complete Question Explanation
(The complete setup for this game can be found here: lsat/viewtopic.php?t=6505)

The correct answer choice is (E)

If Zone 1 has more sales representatives than Zone 3, only one Numerical Distribution is possible: 4-1-2. And, the only template under which a 4-1-2 distribution can occur is Template #2. The inclination at this point may be to draw out the entire solution to Template #2 in a 4-1-2 arrangement, but there is already enough information from the template and the distribution to eliminate the four incorrect answers without wasting the time to draw out the full solution:

Answer choice (A) is incorrect because in a 4-1-2 distribution only T can work in Zone 2.

Answer choice (B) is incorrect because in a 4-1-2 distribution only T can work in Zone 2.

Answer choice (C) is incorrect because in Template #2, P must work in Zone 1.

Answer choice (D) is incorrect because in Template #2, T must work in Zone 2.

Thus, by process of elimination answer choice (E) is proven correct.

Get the most out of your LSAT Prep Plus subscription.

Analyze and track your performance with our Testing and Analytics Package.